Đến nội dung

Hình ảnh

Tìm max, min S = $x^{2}y+y^{2}z+z^{2}x$


  • Please log in to reply
Chủ đề này có 15 trả lời

#1
mylinhvo9997

mylinhvo9997

    Binh nhất

  • Thành viên
  • 34 Bài viết
Cho x,y,z thoã mãn x + y + z = 5 và $x^{2}+y^{2}+z^{2} = 9$. Tìm max,min của :
S = $x^{2}y+y^{2}z+z^{2}x$
Đừng đề ra mục tiêu của mình chỉ vì người khác cho nó là quan trọng vì chỉ có bạn mới biết được điều gì là tốt nhất cho mình.

#2
Mai Xuan Son

Mai Xuan Son

    Vagrant

  • Thành viên
  • 274 Bài viết

Cho x,y,z thoã mãn x + y + z = 5 và $x^{2}+y^{2}+z^{2} = 9$. Tìm max,min của :
S = $x^{2}y+y^{2}z+z^{2}x$

$3S=y(x^2+2yz)+z(y^2+2xz)+x(z^2+2xy)\leq (x^2+y^2+z^2)[(x^2+2yz)^2+(y^2+2xz)^2+(z^2+2xy)^2]$
Từ giả thiết ta đính được cái VP
suy ra $|S|\leq const$
~~~like phát~~~

#3
phuc_90

phuc_90

    Sĩ quan

  • Thành viên
  • 438 Bài viết
Nhìn vào là biết sai rồi ... yêu cầu bạn giải ra rõ ràng nhé . Tìm Max, Min nó khó chứ không dễ đâu bạn.

#4
Mai Xuan Son

Mai Xuan Son

    Vagrant

  • Thành viên
  • 274 Bài viết

Nhìn vào là biết sai rồi ... yêu cầu bạn giải ra rõ ràng nhé . Tìm Max, Min nó khó chứ không dễ đâu bạn.

Nói xem sai ở đâu nào,đừng spam nha
~~~like phát~~~

#5
mchkp2000

mchkp2000

    Binh nhì

  • Thành viên
  • 18 Bài viết

Nói xem sai ở đâu nào,đừng spam nha

dấu bằng có xảy ra được không nhỉ?
bài này rất khó, trong trường hợp x < y < z mình tìm được max là 14 khi x=1, y=z=2 và x<z<y thì min là 248/18 khi x=z=4/3 và y=7/3

#6
Mai Xuan Son

Mai Xuan Son

    Vagrant

  • Thành viên
  • 274 Bài viết

dấu bằng có xảy ra được không nhỉ?
bài này rất khó, trong trường hợp x < y < z mình tìm được max là 14 khi x=1, y=z=2 và x<z<y thì min là 248/18 khi x=z=4/3 và y=7/3

Được đấy bạn ạ
~~~like phát~~~

#7
mchkp2000

mchkp2000

    Binh nhì

  • Thành viên
  • 18 Bài viết

$3S=y(x^2+2yz)+z(y^2+2xz)+x(z^2+2xy)\leq (x^2+y^2+z^2)[(x^2+2yz)^2+(y^2+2xz)^2+(z^2+2xy)^2]$
Từ giả thiết ta đính được cái VP
suy ra $|S|\leq const$

chưa thấy rõ ràng gì cả...const là bao nhiêu và dấu bằng xảy ra khi nào?

#8
mchkp2000

mchkp2000

    Binh nhì

  • Thành viên
  • 18 Bài viết

Cho x,y,z thoã mãn x + y + z = 5 và $x^{2}+y^{2}+z^{2} = 9$. Tìm max,min của :
S = $x^{2}y+y^{2}z+z^{2}x$

sao ko thấy ai giải vậy ta? khó thế nhỉ

#9
mchkp2000

mchkp2000

    Binh nhì

  • Thành viên
  • 18 Bài viết

Cho x,y,z thoã mãn x + y + z = 5 và $x^{2}+y^{2}+z^{2} = 9$. Tìm max,min của :
S = $x^{2}y+y^{2}z+z^{2}x$

khó thật...làm hoài không ra...chỉ làm được hai trường hợp. Bài này ở đâu vậy mylinhvo9997?

#10
mylinhvo9997

mylinhvo9997

    Binh nhất

  • Thành viên
  • 34 Bài viết

khó thật...làm hoài không ra...chỉ làm được hai trường hợp. Bài này ở đâu vậy mylinhvo9997?

bài này có trong quyển sáng tạo bđt của Phạm Kim Hùng bạn à :lol:
Đừng đề ra mục tiêu của mình chỉ vì người khác cho nó là quan trọng vì chỉ có bạn mới biết được điều gì là tốt nhất cho mình.

#11
BoFaKe

BoFaKe

    Thiếu úy

  • Thành viên
  • 613 Bài viết

Cho x,y,z thoã mãn x + y + z = 5 và $x^{2}+y^{2}+z^{2} = 9$. Tìm max,min của :
S = $x^{2}y+y^{2}z+z^{2}x$

Mình nói hướng thế này không biết đúng không(anh em gạch đá mạnh tay để mình tỉnh nhé :wacko: ).Từ cái đk thì mình có thể biểu diễn 2 ẩn $y,z$ qua $x$ (chắc vậy) rồi sau đó thế vào $S$ và coi là hàm số ẩn $x$ với $x\in \left [ -3;3 \right ]$.Từ đó xét đạo hàm và lập bảng biến thiên là tìm được min và max.
P/S:thấy cách này nó ảo ảo thế nào ấy :icon13:
~~~~~~~~~~~~~~Tiếc gì mà không click vào nút like mọi ngươì nhỉ ^0^~~~~~~~~~~~~~

#12
Mushz

Mushz

    Binh nhất

  • Thành viên
  • 41 Bài viết

Mình nói hướng thế này không biết đúng không(anh em gạch đá mạnh tay để mình tỉnh nhé :wacko: ).Từ cái đk thì mình có thể biểu diễn 2 ẩn $y,z$ qua $x$ (chắc vậy) rồi sau đó thế vào $S$ và coi là hàm số ẩn $x$ với $x\in \left [ -3;3 \right ]$.Từ đó xét đạo hàm và lập bảng biến thiên là tìm được min và max.
P/S:thấy cách này nó ảo ảo thế nào ấy :icon13:

giới hạn đc $1\leqslant x,y,z\leq \frac{7}{3}$ lận đó bạn. Mình chưa ra nhưng cũng xin góp thêm chút đỉnh đó.

#13
nthoangcute

nthoangcute

    Thiếu tá

  • Thành viên
  • 2003 Bài viết

Cho x,y,z thoã mãn x + y + z = 5 và $x^{2}+y^{2}+z^{2} = 9$. Tìm max,min của :
S = $x^{2}y+y^{2}z+z^{2}x$

Mình chém bừa cách này không biết có đúng không nữa !!!
Thay $z=5-x-y$ ta được: $$x^2+xy+y^2-5x-5y+8=0\;\;\;\;(C_1)$$
Và $$S=x^3+3x^2y-10x^2-10xy+25x-y^3+5y^2\;\;\;\;\;(C_2)$$
Điều kiện cần và đủ để $S$ có cực trị là hai đường cong $(C_1)$ và $(C_2)$ tiếp xúc với nhau !

__________________________________________
Phần trên là chuẩn xác 100 %
Phần dưới thì khó lý luận nên đành làm liều:
Ta đưa hết về 1 biến !
$$S=S+(x-y)(x^2+xy+y^2-5x-5y+8)=3x^2y+17x-5x^2-10yx+8y$$
$$yS-(3x^2-10x+8)(x^2+y^2+8-5x-5y+yx)=(-41x+20x^2-S+40-3x^3)y+25x^3-82x^2+120x-3x^4-64$$
Do đó hai đường cong:
$$y_1=-{\frac {-25\,{x}^{3}+82\,{x}^{2}-120\,x+3\,{x}^{4}+64}{41\,x-20\,{x}^
{2}+s-40+3\,{x}^{3}}}$$
Và $$y_2={\frac {-17\,x+5\,{x}^{2}+s}{3\,{x}^{2}-10\,x+8}}$$
sẽ tiếp súc với nhau !
Điều này sảy ra khi và chỉ khi $$\left\{\begin{matrix}
y_1=y_2\\
y'_1=y'_2
\end{matrix}\right. $$
Giải hệ phương trình ta được:
$$(S,x)=(\frac{41}{3},-\frac{2}{3} \cos \frac{2 \pi}{9}+\frac{5}{3});(\frac{127}{9}, -\frac{2}{3} \cos \frac{\pi}{9}+\frac{5}{3})$$
cùng các bộ nghiệm khác !
Thực ra $x$ là nghiệm của các phương trình:
$$9x^3-45x^2+72x-37=0\\
(3x)^3-15(3x)^2+72.3x-109=0$$
___________
Cách này hơi khó hiểu, có gì không hiểu cứ hỏi !

Bài viết đã được chỉnh sửa nội dung bởi nthoangcute: 30-12-2012 - 12:06

BÙI THẾ VIỆT - Chuyên gia Thủ Thuật CASIO

 

Facebook : facebook.com/viet.alexander.7


Youtube : youtube.com/nthoangcute


Gmail : [email protected]


SÐT : 0965734893


#14
dark templar

dark templar

    Kael-Invoker

  • Hiệp sỹ
  • 3788 Bài viết
Bài này thì nên tìm cực trị bằng phương pháp Nhân tử Lagrange với hàm Lagrange $L(x;y;\lambda)=f(x;y)+\lambda.g(x;y)$,trong đó $g(x;y)=0$ là hàm điều kiện.
Trong bài toán này thì $g(x;y)=x^2+y^2+xy-5x-5y+8$.
Khi đó hàm Lagrange sẽ là :
$$L(x;y;\lambda)=x^3+3x^2y-10x^2-10xy+25x-y^3+5y^2+\lambda (x^2+y^2+xy-5x-5y+8)$$
Điểm dừng $(\lambda_0;x_0;y_0)$ sẽ là nghiệm của hệ :
$$\left\{\begin{matrix} \frac{\partial L(x;y;\lambda )}{\partial x}=0\\ \frac{\partial L(x;y;\lambda )}{\partial y}=0\\ g(x;y)=0 \end{matrix}\right. \iff \left\{\begin{matrix}3x^2+6xy-20x-10y+25+\lambda(2x+y-5)=0\\ 3x^2-10x-3y^2+10y+\lambda(2y+x-5)=0\\ x^2+y^2+xy-5x-5y+8=0\end{matrix}\right. \quad (*)$$
Giải hệ này sẽ cho ta bộ nghiệm điểm dừng $(x_0;y_0;\lambda_0)$.
Tiếp theo ta xét ma trận Hesse bound :
$$H_{b}=\begin{pmatrix}\frac{\partial ^2L(x;y;\lambda)}{\partial x^2} & \frac{\partial ^2 L(x;y;\lambda)}{\partial x\partial y} & \frac{\partial g(x;y)}{\partial x}\\ \frac{\partial ^2L(x;y;\lambda)}{\partial y\partial x} & \frac{\partial ^2L(x;y;\lambda)}{\partial y^2} & \frac{\partial g(x;y)}{\partial y}\\ \frac{\partial g(x;y)}{\partial x} & \frac{\partial g(x;y)}{\partial y} & 0\end{pmatrix}$$
Thay bộ nghiệm điểm dừng vào ma trận $H_{b}$,nếu $\det H_{b}>0$ thì $f(x;y)$ đạt cực đại tai $(x_0;y_0)$,còn $\det H_{b}<0$ thì đạt cực tiểu.
Vấn đề chỉ là giải hệ (*) mà thôi.Ai giỏi giải hệ thì vô nhé :)
"Do you still... believe in me ?" Sarah Kerrigan asked Jim Raynor - Starcraft II:Heart Of The Swarm.

#15
mchkp2000

mchkp2000

    Binh nhì

  • Thành viên
  • 18 Bài viết

Cho x,y,z thoã mãn x + y + z = 5 và $x^{2}+y^{2}+z^{2} = 9$. Tìm max,min của :
S = $x^{2}y+y^{2}z+z^{2}x$

ý tưởng của mình:
Ta có $(x+y+z)^{2}=25\Rightarrow xy+yz+zx=8$
$\Rightarrow xyz=8z-z^{2}(x+y)=8z-z^{2}(5-z)=z^{3}-5z^{2}+8z$
và $x^{3}+y^{3}+z^{3}=(x+y+z)(x^{2}+y^{2}+z^{2}-xy-yz-zx)+3xyz=5+3xyz$
lại có $y+z=5-x$ và $yz=x^{2}-5x+8$
dùng điều kiện có nghiệm ta được $ x, y,z \in [1,7/3]$
TH1: $x\leq y\leq z\Rightarrow z\geq 4/3$
khi đó
$S\leq xy^{2}+yz^{2}+zx^{2}$
$\Rightarrow 2S\leq (x+y+z)(x^{2}+y^{2}+z^{2})-(x^{3}+y^{3}+z^{3})=45-(5+3xyz)=-3z^{3}+15z^{2}-24z+40=f(z)$
khảo sát $f(z)$ trên $[4/3,7/3]$ ta được
$maxf=28$ khi $z=2$ suy ra $maxS=14$ khi $z=y=2,x=1$
còn min chưa tìm được... :(
TH2: $x\leq z\leq y \Rightarrow x< 2 $
tương tự như trên ta có
$2S\geq 45-((x^{3}+y^{3}+z^{3}))=-3x^{3}+15x^{2}-24x+40=f(x)$
khảo sát f trên $[1,2]$ ta được
$minf=248/9$ hay $minS=248/18$ khi $x=z=4/3,y=7/3$
trường hợp này chưa tìm được max.... :biggrin:

#16
Mai Xuan Son

Mai Xuan Son

    Vagrant

  • Thành viên
  • 274 Bài viết

 

 

 

 

Các bạn hình như sai rồi, nếu dùng $Lagrange$ đầu tiên phải có động thái sau 

Đặt $\left\{\begin{matrix} x=a+\frac{5}{3} & & \\ y=b+\frac{5}{3} & & \\ z=c+\frac{5}{3} & & \end{matrix}\right.$

Suy ra $\left\{\begin{matrix} a+b+c=0 & \\ a^2+b^2+c^2=\frac{2}{3} & \end{matrix}\right.$

Bài toán quy về tìm $\max$ và $\min$ của $P=a^2b+b^2c+c^2a+\frac{125}{9}$

Theo $Lagrange$ ta tìm được điểm rơi tại hệ

$\frac{x^2+2yz}{y}=\frac{y^2+2xz}{z}=\frac{z^2+2xy}{x}$

Viết lại $P$ như sau $(3P)^2=[a.(a^2+2bc)+c.(b^2+2ac)+a.(c^2+2ab)]^2\leq[a^2+b^2+c^2].[\sum _{cyc}(a^2+2bc)] =const$

Đạt cực trị tại $(a,b,c)\sim (t.\cos\frac{2\pi }{9};t.\cos\frac{4\pi }{9},t.\cos\frac{8\pi }{9})$

$t$ thì chỉ cần chọn thôi $\blacksquare$


~~~like phát~~~




1 người đang xem chủ đề

0 thành viên, 1 khách, 0 thành viên ẩn danh